2022 AMC 10B Problems/Problem 17
Problem
One of the following numbers is not divisible by any prime number less than 10. Which is it?
Solution
For A, modulo 3,
Thus, is divisible by 3.
For B, modulo 5,
Thus, is divisible by 5.
For D, modulo 3,
Thus, is divisible by 3.
For E, module 5,
Thus, is divisible by 5.
Therefore, the answer is $\boxed{\textbf{(C) 2^{607} - 1}}$ (Error compiling LaTeX. Unknown error_msg).
~Steven Chen (Professor Chen Education Palace, www.professorchenedu.com)
~MrThinker (LaTeX Error)
Video Solution
~Steven Chen (Professor Chen Education Palace, www.professorchenedu.com)